Περίεργη συντρέχεια

Συντονιστές: vittasko, silouan, rek2

min##
Δημοσιεύσεις: 342
Εγγραφή: Τρί Απρ 18, 2017 3:40 pm

Περίεργη συντρέχεια

#1

Μη αναγνωσμένη δημοσίευση από min## » Κυρ Ιουν 09, 2019 7:32 pm

Ένα θέμα που μου προέκυψε (μπορεί να υπάρχει πολύ άμεση λύση,μπορεί και όχι):Έστω τρίγωνο ABC και A',B',C' τα αντιδιαμετρικά των κορυφών στον περίκυκλο.Έστω X,Y τα μέσα των τόξων AC,AB και S,T οι τομές της ευθείας που ενώνει το περίκεντρο με το έκκεντρο του τριγώνου,με τις AB,AC.Αν l η εφαπτομένη στον AST στο A και R η τομή της με τον (ABC) νδο. A'R,B'C',XY συντρέχουν.



Λέξεις Κλειδιά:
Άβαταρ μέλους
vittasko
Επιμελητής
Δημοσιεύσεις: 2230
Εγγραφή: Πέμ Ιαν 08, 2009 8:46 am
Τοποθεσία: Μαρούσι - Αθήνα.
Επικοινωνία:

Re: Περίεργη συντρέχεια

#2

Μη αναγνωσμένη δημοσίευση από vittasko » Δευ Ιουν 10, 2019 1:02 am

Έστω το σημείο K\equiv XY\cap IO, όπου I,\ O είναι το έγκεντρο και το περίκεντρο αντιστοίχως, του δοσμένου τριγώνου \vartriangle ABC.

\bullet Αποδεικνύεται ( η απόδειξη αφήνεται στον αναγνώστη, ως άσκηση στα αρμονικά συζυγή ) ότι ισχύει (KA)^{2} = (KS)(KT)\ \ \ ,(1) και επομένως, έχουμε ότι η ευθεία KA εφάπτεται στον περίκυκλο του τριγώνου \vartriangle AST και άρα, R\equiv (O)\cap AK.

Έστω D, το σημείο επαφής του έγκυκλου (I) του \vartriangle ABC στην πλευρά BC και έστω το σημείο L\equiv XY\cap ID.

Τα σημεία A,\ L,\ O, είναι συνευθειακά ( γνωστό αποτέλεσμα που αποδεικνύεται εύκολα ).

Έστω τα σημεία M\equiv AK\cap DL και N\equiv A'B'\cap MX.

Αποδεικνύεται ( όχι δύσκολα και θα ακολουθήσει το σχετικό Λήμμα) ότι ισχύει MN\perp A'B'.
f 181_t 64631.PNG
Περίεργη συντρέχεια.
f 181_t 64631.PNG (43.88 KiB) Προβλήθηκε 1237 φορές
\bullet Παρατηρούμε τώρα, ότι οι δια των κορυφών A,\ M,\ I του τριγώνου \vartriangle AMI κάθετες ευθείες, επί των ευθειών των πλευρών A'X,\ A'B',\ B'X αντιστοίχως, του τριγώνου \vartriangle B'XA', συντρέχουν στο σημείο X ( AX\perp XA' και MX\perp A'B' και IX\perp B'X ) και άρα, τα τρίγωνα \vartriangle AMI,\ \vartriangle B'XA' είναι ορθολογικά.

Συμπεραίνεται έτσι, ότι οι ευθείες XY,\ B'C',\ A'X, ως οι δια των κορυφών X',\ B',\ A' κάθετες ευθείες επί των ευθειών των πλευρών AI,\ MI,\ AM αντιστοίχως, του τριγώνου \vartriangle AMI ( XY\perp AI και B'C'\perp MI\equiv ID λόγω B'C'\parallel BC και A'R\perp AR\equiv AM ), τέμνονται στο ίδιο σημείο έστω Z και το ζητούμενο έχει αποδειχθεί.

Κώστας Βήττας.

ΥΓ. Θα βάλω από αύριο το σχήμα και τις λεπτομέρειες για το Λήμμα που αναφέρω πιο πάνω και για την (1) αργότερα, αν δεν απαντηθεί.


min##
Δημοσιεύσεις: 342
Εγγραφή: Τρί Απρ 18, 2017 3:40 pm

Re: Περίεργη συντρέχεια

#3

Μη αναγνωσμένη δημοσίευση από min## » Δευ Ιουν 10, 2019 1:16 pm

Εξαιρετική λύση, το χτυπάει στην "καρδιά" του. :coolspeak:


Άβαταρ μέλους
vittasko
Επιμελητής
Δημοσιεύσεις: 2230
Εγγραφή: Πέμ Ιαν 08, 2009 8:46 am
Τοποθεσία: Μαρούσι - Αθήνα.
Επικοινωνία:

Re: Περίεργη συντρέχεια

#4

Μη αναγνωσμένη δημοσίευση από vittasko » Τρί Ιουν 11, 2019 11:51 pm

Μίνο, σ' ευχαριστώ πολύ. Eύχομαι σε όλους σας καλή επιτυχία στην ΙΜΟ.
vittasko έγραψε:
Δευ Ιουν 10, 2019 1:02 am
...Αποδεικνύεται ( όχι δύσκολα και θα ακολουθήσει το σχετικό Λήμμα) ότι ισχύει MN\perp A'B'.
ΛΗΜΜΑ. - Δίνεται τρίγωνο \vartriangle ABC με περίκυκλο (O), έγκυκλο (I) και ας είναι X,\ Y, τα σημεία τομής του (O) από τις ευθείες BI,\ CI αντιστοίχως, με I το κέντρο του (I). Έστω τα σημεία K\equiv XY\cap IO και M\equiv AK\cap ID και N\equiv A'B'\cap MX, όπου O είναι το κέντρο του (O) και D, το σημείο επαφής του (I) στην BC και A',\ B', τα αντιδιαμετρικά σημεία των A,\ B, αντιστοίχως. Αποδείξτε ότι MX\perp A'B'.

\bullet Η ευθεία XY ταυτίζεται με την μεσοκάθετη ευθεία του AI ( γνωστό αποτέλεσμα που αποδεικνύεται εύκολα ) και έστω το σημείο Q\equiv AI\cap XY.

Άρα, οι ευθείες KA,\ KO είναι συμμετρικές ως προς την ευθεία XY.

Συμμετρικές ως προς την XY είναι επίσης και οι ευθείες LA,\ LI, όπου L\equiv XY\cap ID και επομένως, ισχύει \angle LAX = \angle LIX\ \ \ ,(1)

Αλλά, \angle LIX = \angle DIB = 90^{o} - \angle DBX = 90^{o} - \angle ABX = 90^{o} - \angle AA'X\ \ \ ,(2)

Από (1),\ (2)\Rightarrow \angle LAX = \angle A'AX\Rightarrow AL\equiv AA'\ \ \ ,(3) και άρα, τα σημεία A,\ L,\ O είναι συνευθειακά και οι ευθείες AO,\ IM είναι συμμετρικές ως προς την XY.
f 181_t 64631(a).PNG
Περίεργη συντρέχεια - Απόδειξη του Λήμματος.
f 181_t 64631(a).PNG (26.83 KiB) Προβλήθηκε 1190 φορές
\bullet Προκύπτει έτσι, ότι και οι ευθείες XM,\ XO είναι επίσης συμμετρικές ως προς την XY και άρα, έχουμε \angle MXQ = \angle OXQ\ \ \ ,(4)

Αλλά, ισχύει \displaystyle \angle OXQ = \frac{\angle A}{2}\ \ \ ,(5) λόγω του εγγραψίμου τετραπλεύρου AXWQ, όπου W\equiv AC\cap OX.

Από (4),\ (5)\Rightarrow \displaystyle \angle MXQ = \frac{\angle A}{2}\ \ \ ,(6)

Από (6) έχουμε ότι το τετράπλευρο APXQ, όπου P\equiv MX\cap AB είναι εγγράψιμο και άρα, προκύπτει \angle APX = 90^{o}, λόγω \angle AQX = 90^{o} και άρα, έχουμε MX\perp AP\equiv AB\ \ \ ,(7)

Από (7) και A'B'\parallel AB, συμπεραίνεται ότι MX\perp A'B' και το Λήμμα έχει αποδειχθεί.

Κώστας Βήττας.


Άβαταρ μέλους
Διονύσιος Αδαμόπουλος
Δημοσιεύσεις: 807
Εγγραφή: Σάβ Μαρ 19, 2016 5:11 pm
Τοποθεσία: Πύργος Ηλείας

Re: Περίεργη συντρέχεια

#5

Μη αναγνωσμένη δημοσίευση από Διονύσιος Αδαμόπουλος » Τετ Ιουν 12, 2019 1:08 pm

Ωραίο πρόβλημα! Θα πατήσω λίγο στην λύση του κύριου Βήττα και συγκεκριμένα στο "ζωτικής σημασίας" σημείο K.

Έστω πως οι B'Y και C'X τέμνονται στο A''. Προφανώς το A'' προκύπτει να είναι το αντιδιαμετρικό του I στο τρίγωνο XIY.

Όμως στη συμμετρία ως προς την XY, τα A, I είναι συζυγή, όπως και τα τρίγωνα XAY και XIY, άρα τα A' και A'' είναι συζυγή.

To R που το ορίζουμε εκ νέου ως την τομή της KA και του περιγεγραμμένου κύκλου του AXY, έχει συζυγές το R', όπου είναι η τομή της KI, δηλαδή της OI, με την XY.

Έστω U το σημείο τομής της XY και B'C'.

Αρκεί να δείξουμε πως τα A'', U, R' είναι συνευθειακά.

Έστω τώρα W το σημείο τομής της C'Y και B'X.

Από Pascal αυτό βρίσκεται πάνω στην OI.

Παρατηρούμε πως η A''U συνιστά την πολική του W στον περιγεγραμμένο κύκλο του ABC.

Θα δείξουμε ότι και το R' ανήκει στην πολική του W.

Έστω πως η OI τέμνει τον περιγεγραμμένο στα Z, Q.

Επειδή το O είναι μέσο του ZQ, αρκεί από Maclaurin: WZ\cdot WQ=WR'\cdot WO\Leftrightarrow WC'\cdot WY=WR'\cdot WO.

Αρκεί δηλαδή C'YOR' να είναι εγγράψιμο.

Έχουμε: \widehat{OR'Y}=\widehat{IR'Y}=\widehat{IXY}=\widehat{YBC}=\widehat{C'BC}-\widehat{C'BY}=90^o-\dfrac{\widehat{C'OY}}{2}=\widehat{OC'Y}

---------------------------------------------------------------------------------------------------------
Θα δείξω τώρα γιατί η KA είναι εφαπτομένη στο AST (η αρχή της λύσης του κύριου Βήττα)

Έστω πως η XY τέμνει τις AS και AT στα D, E.

Από Μενέλαο έχουμε:

\dfrac{KS}{KT}\dfrac{DA}{DS}\dfrac{ET}{EA}=1

Όμως εύκολα συμπεραίνουμε πως το ADIE είναι ρόμβος.

Άρα \dfrac{DA}{DS}=\dfrac{IT}{SI} και \dfrac{ET}{EA}=\dfrac{IT}{SI}, άρα \dfrac{KS}{KT}=\dfrac{SI^2}{IT^2} και από θεώρημα διχοτόμων στο AST, έχουμε πως \dfrac{KS}{KT}=\dfrac{AS^2}{AT^2} και ως γνωστό έπεται ότι η KA είναι εφαπτομένη στο AST.


Houston, we have a problem!
Απάντηση

Επιστροφή σε “Γεωμετρία - Επίπεδο Αρχιμήδη (Seniors)”

Μέλη σε σύνδεση

Μέλη σε αυτήν τη Δ. Συζήτηση: Δεν υπάρχουν εγγεγραμμένα μέλη και 16 επισκέπτες